- PowerScore Staff
- Posts: 5972
- Joined: Mar 25, 2011
- Tue Oct 23, 2018 4:00 pm
#59781
Complete Question Explanation
(The complete setup for this game can be found here: lsat/viewtopic.php?t=1655)
The correct answer choice is (C)
The question stem asserts that only one of the five sites dates from the 10th century, and because we have already established that the third site dates from the 10th century, we know that the third site is the only site from the 10th century. In addition, from the third rule we know that G must discover a site from the 10th century, and so we can conclude that G must discover the third site. In this Cannot Be True question, then, F cannot discover the third site and answer choice (C) is correct.
Note that although other deductions could be made (such as that the fifth site dates from the 9th century), and an entire diagram could be drawn out for this question, there is no point in doing so since our initial diagram already determined that the third site was from the 10th century, and that provides the starting point for a fast and clear path to the correct answer.
Answer choices (A), (B), (D), and (E) are each incorrect because F could discover any of the sites listed in these answer choices.
(The complete setup for this game can be found here: lsat/viewtopic.php?t=1655)
The correct answer choice is (C)
The question stem asserts that only one of the five sites dates from the 10th century, and because we have already established that the third site dates from the 10th century, we know that the third site is the only site from the 10th century. In addition, from the third rule we know that G must discover a site from the 10th century, and so we can conclude that G must discover the third site. In this Cannot Be True question, then, F cannot discover the third site and answer choice (C) is correct.
Note that although other deductions could be made (such as that the fifth site dates from the 9th century), and an entire diagram could be drawn out for this question, there is no point in doing so since our initial diagram already determined that the third site was from the 10th century, and that provides the starting point for a fast and clear path to the correct answer.
Answer choices (A), (B), (D), and (E) are each incorrect because F could discover any of the sites listed in these answer choices.
Dave Killoran
PowerScore Test Preparation
Follow me on X/Twitter at http://twitter.com/DaveKilloran
My LSAT Articles: http://blog.powerscore.com/lsat/author/dave-killoran
PowerScore Podcast: http://www.powerscore.com/lsat/podcast/
PowerScore Test Preparation
Follow me on X/Twitter at http://twitter.com/DaveKilloran
My LSAT Articles: http://blog.powerscore.com/lsat/author/dave-killoran
PowerScore Podcast: http://www.powerscore.com/lsat/podcast/